El problema al que me enfrento es el del título:
El problema. Demostrar que $\mathbb{Q}(\sqrt{23})$ no es un campo numérico euclidiano.
Desde $23\not\equiv 1\pmod{4}$ hay que demostrar que $\mathbb{Z}[\sqrt{23}]$ no es un dominio euclidiano.
Sé cómo demostrar que no es norma euclidiana, pero aún así podría ser euclidiana con una función diferente.
La única forma que conozco de demostrar que un dominio no es euclidiano es utilizando el teorema de Motzkin:
Teorema. Un dominio $D$ es euclidiano si y sólo si $\bigcap\limits_{i\in\mathbb{N}} P_0^{(i)} = \emptyset$ .
Aquí $P_0^{(0)} = P_0 := D\setminus\{0\}$ y definimos recursivamente $P^{(i+1)}_0 := (P^{(i)}_0)'$ para todos $i\in\mathbb{N}$ donde para cualquier $P\subseteq D$ el derivado set $P'$ de $P$ viene dada por $P' := \{b\in P\, ;\, \exists a\in D\text{ such that }a+bD\subseteq P\}$ .
Funciona bien para los campos cuadráticos imaginarios ya que sólo tienen $1$ y $-1$ como unidades: Considerando $K := \mathbb{Q}(\sqrt{d})$ para $d < -11$ se puede demostrar que $P_0^{(i)} = A\setminus\{0, 1, -1\}$ para todos los enteros $i\ge 1$ , donde $A$ es el anillo de enteros de $K$ Por lo tanto, por el teorema de Motzkin $K$ no es euclidiano. Pero en $\mathbb{Q}(\sqrt{23})$ tenemos demasiadas unidades y ni siquiera puedo determinar $P_0''$ .
0 votos
Ver también mathoverflow.net/questions/51503/ .
0 votos
No eres el único que está perplejo ante dominios como estos: math.stackexchange.com/questions/1148364/ De hecho, 22, 31, 43, 46, 47, 53, 59, 61, 62, 67, 71 también parecen ser euclidianas pero sin función euclidiana conocida.
1 votos
Existe una conjetura que afirma que un campo numérico $K$ es euclidiano si es un PID (por supuesto, $K$ no debe ser de la forma $\mathbb{Q}(\sqrt{n})$ para $n<0$ ). Ahora bien, se sabe que $\mathbb{Q}(\sqrt{n})$ es euclidiano si es un PID para, al menos, $n\leq 500$ . Como el número de clase cuadrático de 23 es 1, 23 es un PID y, por tanto, euclidiano.
4 votos
Si consigue demostrar el enunciado, habrá refutado la conjetura de Riemann para al menos una función L (véase Weinberger). Buena suerte.
2 votos
Ya lo dijeron Ángel Valencia y Franz Lemmermeyer: Deberías leer "On Euclidean Rings Of Algebraic Number Fields" de Weinberg. Él fue capaz de demostrar (¡con GRH!), que si el anillo de enteros de un campo numérico real-cuadrático es un PID, también debe ser euclidiano. Por tanto, $\mathbb{Q}[\sqrt{23}]$ probablemente no será euclidiano. Por otro lado: En el caso de que pudieras demostrar tu "Problema", deberías ser capaz de demostrar, que la GRH no es cierta. $\rightarrow$ Dinero dinero dinero!! ;)